Need help ASAP DUE AOLN AND PLEASE SHOW WORK ?!!!

Need Help ASAP DUE AOLN AND PLEASE SHOW WORK ?!!!

Answers

Answer 1

Answer: 35.98in

Step-by-step explanation:

Use the formula: h=3 v/pi(r^2)

Answer 2

Answer:

V = 36.0 in

Step-by-step explanation:

V of cylinder = (1/3)πr^2(h)

V of cylinder = 3052 cubic inches

r = 9 in

3052 = (1/3)π(9)^2(h)

3052 = (1/3)π(81)h

3052 = 84.82300165h

divide both sides by coefficient of h

h = 36.0 in (rounded to nearest tenth)


Related Questions

You earn $30 for washing 5 cars. How much do you earn
per car?

Answers

Answer:

You get 6 dollars per car

Step-by-step explanation:

I divided 30 by 5

Answer: $6 per car

Step-by-step explanation:

If it's 30 dollars for 5 cars and we want to know how much it is for 1 car we need to divide both by 5

[tex]\frac{30}{5} =6\\\frac{5}{5} =1[/tex]

$6 per one car

What should be the first step in adding these equations to eliminate y?

Answers

Answer:

D

Step-by-step explanation:

Answer:

D

Step-by-step explanation:

Since you want to eliminate y, the y value on the bottom equation has the be the opposite of the y value on the top one. So it's 4y on the top one, so the bottom one has to be -4y to eliminate y. The bottom one is -2y, so -2 times 2 =-4, so multiply the bottom one by 2 omg i miss linear combination

Select the correct answer.

Simplify the expression 5^-3 x 5^-5

A 1/5^2

B -5^2

C 5^2

D 1/5

Answers

Answer:

A 1/5^2

Step-by-step explanation:

5^3 x 5^-5

We know that a^b* a^c = a^(b+c)

5^3 x 5^-5 = 5^(3-5)  = 5^-2

We also know that a^-b = 1/ a^b

5^-2 = 1/5^2

A. 6
B. 2
C. 0.4
D. -6

Answers

Answer:

6

Step-by-step explanation:

Hope thats the correct answer.

A rectangular flower pot is 30 cm long, 16 cm wide and 12 cm high. How much soil is needed to fill the flower pot?

Answers

5,760 cubic centimeters of soil is needed to fill the rectangular flower pot

This question is asking for the volume of a rectangular prism (length x width x height):
30 • 16 • 12 = 5,760

The volume is measured in cubic feet in this case.

Final answer: 5,760 cubic feet of soil

Hope this helps!

which of the following is equivalent to 2cos^2(x) - cos(2x)? A. sin(2x) B. Sin^2(x) C. 1 D. 2

Answers

Answer:

C. 1

Step-by-step explanation:

2cos²(x) - cos(2x) = 1

Answer:

C

Step-by-step explanation:

Using the identity

cos2x = 2cos²x - 1 , then

2cos²x = cos2x + 1

Given

2cos²x - cos2x

= cos2x + 1 - cos2x

= 1 → C

THERE ARE 18 RED CRAYONS
AND 24 GREEN CRAYONS IN A
BASKET. WHAT IS THE RATIO OF
RED TO GREEN CRAYONS

Answers

Answer:

4 : 3

Step-by-step explanation:

So to answer this, we must find the greatest common factor(GCF) of 18 adn 24 , then divide by that to get the ratio.

So, what are the factors of 18?

2369118

Now what are the factors of 24?

2368124

All of these work, but we need to GCF, which is the largest factor that goes into both numbers.

Looking at them, it would be 6.

So lets divide both numbers by 6 and see what we get:

18/6

=

3

24/6

=

4

So now there are 3 red crayons and 4 green crayons.

OR

For every 3 red crayons there are 4 green crayons.

This makes the ratio:

3 : 4

Hope this helps!

Ration of red to green crayons is 3:4

Given that;

Number of red crayons = 18

Number of green crayons = 24

Find:

Ration of red to green crayons

Computation:

Ration of red to green crayons = Number of red crayons / Number of green crayons

Ration of red to green crayons = 18 / 24

Ration of red to green crayons = 3/4

Ration of red to green crayons = 3:4

Learn more:

https://brainly.com/question/20753385?referrer=searchResults

6 friends go out for dinner and the bill for food and drink comes to £140.
The friends decide to add a 20% tip on top of this total.
If they divide the expense equally between them, how much does each friend pay?

Answers

Answer:

£28

Step-by-step explanation:

140 x .2 = 28

28+140 = 168

168/6 = 28

One integer is 10 more than another, Their sum is 4. What are the 2 integers? PLEASE HELP!

Answers

Answer:

The two integers are : -3 , 7

Step-by-step explanation:

Let the integers be x and y

Given one of the integer is 10 more than the other, Let x = y + 10 -----(1)

Also Given sum of integers . x + y = 4 ----------- (2)

Substitute ( 1 ) in (2)

x + y = 4

(y + 10) + y = 4

y + 10 + y = 4

2y + 10 = 4

2y = 4 - 10

2y = -6

y = -3

Substitute y = -3 in ( 1 ) : x = - 3 + 10 = 7

What is the domain of the function on the graph

Answers

Answer:

The domain is all x-values or inputs of a function. hope this helps!

Step-by-step explanation:

Find the sum of the numbers. Express your answer in scientific notation.
(6.94 x 10-7)+(2.7x10-)
O 7.21x 10-
O 9.64 x 10-
O 7.21x10-2
O 9.64 x 10-7

Answers

Answer:

7.21 × 10⁻⁷

Step-by-step explanation:

(6.94 × 10⁻⁷) + (2.7 × 10⁻⁸) = 7.21 × 10⁻⁷

find the length of side x in simplest radical form with a rational denominator.

Answers

Answer:

[tex]3\sqrt{3}[/tex]

Step-by-step explanation:

In all 30-60-90 triangles, the side lengths are in the ratio [tex]x:x\sqrt{3}:2x[/tex], where [tex]x[/tex] is the side opposite to the 30 degree angle and [tex]2x[/tex] is the hypotenuse of the triangle. Since the side opposite to the 30 degree angle is marked as 3, the value of [tex]x[/tex] must be [tex]\boxed{3\sqrt{3}}[/tex].

Alternatively, we can use basic trig. for a right triangle to solve. In any right triangle, the tangent of an angle is equal to its opposite side divided by its adjacent side. Thus, we have:

[tex]\tan 60^{\circ}=\frac{x}{3},\\x=3\tan 60^{\circ}=\boxed{3\sqrt{3}}[/tex]

Answer:

Step-by-step explanation:

The tangent of a 60 degree angle = Sin(60) / Cos(60)

Sin(60) = Square root (3) / 2

Cos(60) = 1/2

Tan(60) = √3 / 2 // 1/2   Invert the denominator and multiply

Tan(60) = √3/2 * 2/1

Tan(60) = √3

Tan(theta) = opposite / adjacent

Tan(theta) = x/3 in this case

Tan(60) = √3 = x/3

x = 3*√3

A rectangular room is twice as long as its breadth and its perimeter is 48 m. Find the cost of carpeting its floor at Rs. 75 per sq.m. ​

Answers

If the breadth is x and the length is 2x, we know that the perimeter is 2x + x + 2x + x = 48. Which means 6x = 48 so x = 8

Now we can find the surface area because the length is 2x so we substitute the 8 in and we get the length to be 16, and the breadth to be 8 (because breadth = x and x = 8)

Total area = length x width = 16 x 8 = 128

Enter the coordinates of the point on the unit circle at the given angle. 135° (-VII, VI Enter​

Answers

related angle is 45 degrees, set up a triangle and use exact trig values to solve for x and y, then rationalise the denominators

The coordinates of the point on the unit circle at the given angle. 135° is (-√2 / 2, √2 / 2).

What is a unit circle?

A circle on the Cartesian Plane with a radius of one unit and a center position is referred to as a unit circle (0, 0). When working with angles and trigonometric functions, the unit circle is a useful tool that makes reference much simpler. The Pythagorean Theorem can be used with the unit circle.

Given:

The unit circle at the given angle 135°,

Calculate the value of sin45 as shown below,

sin (180 - 135) = 1 / √2

sin 45 = 1 / √2

cos(180 - 135) =  1 / √2

cos45 = 1 / √2

Write the coordinates as shown below,

Coordinates = (1 / √2 × √2 / √2, 1 / √2 × √2 / √2 )  (Second quadrant is (-x, y))

Coordinates = (-√2 / 2, √2 / 2)

Thus, the coordinates are  (-√2 / 2, √2 / 2).

To know more about unit circle:

https://brainly.com/question/11743473

#SPJ2

Complete the Steps to Solve the Equation -13x-11=-37

-13x-11=-37
-13x=

Answers

Uhhmmm sorry to my friend I have to go

Answer:

x=2

Step-by-step explanation:

-13x-11=-37                             Original equation

-13x=-26                                 Add 11 to both sides

x=2                                         Divide both sides by -13

Graph h(x)=-x^4 +3
Use the parabola tool then choose the vertex followed by one point on the parabola.

Answers

Hope this helps, the y intercept is 3 ans the x intercepts are +1.3 and -1.3

The y intercept is 3 and the x intercepts are +1.3 and -1.3

The graph is given below.

What is graph?

In mathematics, the graph of a function f is the set of ordered pairs, where {\displaystyle f(x)=y.} In the common case where x and f(x) are real numbers, these pairs are Cartesian coordinates of points in two-dimensional space and thus form a subset of this plane.

Here, we have,

h(x)=-x^4 +3

so, The y intercept is 3 and the x intercepts are +1.3 and -1.3

The graph is given below.

To learn more on graph click:

brainly.com/question/17267403

#SPJ2

6 of 10
?
What fraction of an hour is 51 minutes?
Give your answer in its simplest form.

Answers

51/60
We need to make sure that the United sets the same so there are 60min in one hour and the fraction of time that is In an hour in 51 so
51/60
51 is prime so it’s can’t be simplifyied

Give m || n, find the value of x and y

Answers

Answer:

Step-by-step explanation:

2x + 4 + 7x - 13 = 180 degree

9x - 9 = 180

9x = 180 + 9

x = 189/9

x = 21 degree

2x + 4 + 3y + 14 = 180 degree (being linear pair)

2*21 + 4 + 3y + 14 = 180

42 + 18 + 3y = 180

60 + 3y = 180

3y = 180 - 60

y = 120/3

y = 40 degree

A 3-column table with 5 rows. Column 1 is labeled Tickets with entries 2, 4, 6, 8, 10. Column 2 is labeled Total Cost for Aquarium with entries 29, 58, 87, blank, blank. Column 3 is labeled Total Cost for Wave Pool with entries 33.5, 67, blank, blank, blank.
Gale found out that the wave pool has a money-saving deal: if more than 4 people attend, the tickets are discounted from $16.75 each to $12.25 each.

Calculate to determine how much 6 tickets to the wave pool will cost.

$

Answers

Answer

What event will cost the least for 9 tickets?

the wave pool

How much less will this event cost?

$20.25

Hope it helps :)

Step-by-step explanation:

Find the volume. It’s a cilinder
6 in.
10 in

Answers

Answer:1130.97

Step-by-step explanation:

V=πr2h=π·62·10≈1130.97336

7x+40
5x+20
Find the value of x.

Answers

Answer:

x = - 10

Step-by-step explanation:

7x + 40 = 5x + 20

7x + 40 - 40 = 5x + 20 - 40

7x = 5x - 20

7x - 5x = 5x - 5x - 20

2x = - 20

2x ÷ 2 = - 20 ÷ 2

x = - 10

Which of the following is most likely the next step in the series?

Answers

⁺˚*・༓☾✧.* ☽༓・*˚⁺‧

❀ [tex]\huge\underline{ \underline{ \text{Answer :-}}}[/tex]

➺ It's likely to be option D.

[tex]\underbrace{ \overbrace{ \mathfrak{Explanation}}}[/tex]

➺ The first figure has a line which divides the circle ⭕ into 2 parts. The 2nd figure divides the circle ⭕ into 3 parts while the 3th figure divides the circle ⭕ into 4 parts. Obviously, the 4th figure then should divide the circle ⭕ into 5 parts. Checking all the given options,

Option A. doesn't divide the circle ⭕ into any parts.Option B. divides the circle ⭕ into 3 partsOption C. divides the circle ⭕ into 4 parts.Option D. divides the circle ⭕ into 5 parts.

⟹ So, option D. is the correct answer.

⁺˚*・༓☾✧.* ☽༓・*˚⁺‧

ʰᵒᵖᵉ ⁱᵗ ʰᵉˡᵖˢ ツ

꧁❣ ʀᴀɪɴʙᴏᴡˢᵃˡᵗ2²2² ࿐

How much growth or decay is 50(1.05)^t<100

Answers

Answer:

5% growth.

Step-by-step explanation:

1.05 = 1 + 0.05 = 1 + 5%

It is 5% growth.

Q= 4+0.8p what value of p would make the equation true if q was 70

Answers

Answer:

82.5 = p

Step-by-step explanation:

Q= 4+0.8p

Let q = 70

70= 4+0.8p

Subtract 4 from each side

70-4= 4-4+0.8p

66 = .8p

Divide each side by .8

66/.8 = .8p/.8

82.5 = p


solving system by substitution

Answers

Given the system of equations below:

[tex] \large{ \begin{cases} x + 2y = 12 \\ x = y - 12 \end{cases}}[/tex]

For the second equation, x-term is isolated and can be substituted in the first equation.

[tex] \large{(y - 12) + 2y = 12}[/tex]

The equation above is when we substitute x = y-12 in the first equation. Cancel the brackets.

[tex] \large{y - 12 + 2y = 12}[/tex]

Add up the like term and isolate y-term.

[tex] \large{3y - 12= 12} \\ \large{3y - 12 + 12 = 12 + 12}[/tex]

Add both sides by 12 to get rid of 12 from the left side to isolate y-term.

[tex] \large{3y = 24}[/tex]

Divide both sides by 3 so we can finally isolate the term.

[tex] \large{ \frac{3y}{3} = \frac{24}{3} } \\ \large{ \frac{ \cancel{3}y}{ \cancel{3}} = \frac{ \cancel{24}}{ \cancel{3}} } \\ \large{y = 8}[/tex]

Next, find the x-value because in system of equations - we have to answer as in an ordered pairs or coordinate point. We know y-value now but we don't know x-value yet. To find x-value, we substitute the y-value in one of two equations that are given. You can substitute in both equation but it's not necessary to substitute in both equations at one. I will choose to substitute in x = y-12.

[tex] \large{x = y - 12}[/tex]

Substitute y = 8 in the equation.

[tex] \large{x = 8 - 12} \\ \large{x = - 4}[/tex]

Now that we know the both values. We finally have an answer to this problem. Hence.

Answer

x = -4, y = 8(-4,8)

The second answer is in ordered pair form. Let me know if you have any doubts!

PLEASE HELP QUICKLY - ATTACHED BELOW MATHS

Answers

Answer:

113.081 mm²

Step-by-step explanation:

A semicircle is the half part of a circle. And we know that the semicircle have 24 mm of diameter, and the radius is 24/2 = 12 mm. The small circle is inside the semicircle, so its diameter is equal to the radius of the semicircle, and its radius is 12/2 = 6mm

Now, consider π = 3.141, the area of the small circle is:

π•6² = 3.141 • 36 = 113.076 mm²

The area of the semicircle is (π•12²)/2 = (3.141•144)/2 = 226.151 mm²

Now, you just subtract the areas:

226.151 - 113.076 = 113.081 mm²

Three friends go to the movies and spend a total of $75.00. Each friend bought a ticket and a small popcorn that cost $8.25 how much was each ticket

Answers

Answer:

16.75 Dollars for each Ticket

Step-by-step explanation:

They are three friends so the total money they spent is 75 Dollars so a popcorn costs 8.25 EACH friend so

8.25 x 3

= 24.75

24.75 dollars was spent on the 3 popcorns.

Subtract the total money spent and the total money spent on the 3 popcorns so

75. 00 - 24.75

= 50.25

50.25 dollars were totally spent on the tickets but we're looking for each ticket so divide it by 3.

50.25/3

= 16.75

16.75 dollars each were spent on each ticket.

Thank you and pls mark me as brainliest! Love y'all!

In ΔUVW, the measure of ∠W=90°, WV = 24, UW = 7, and VU = 25. What is the value of the sine of ∠U to the nearest hundredth?

Answers

Answer:

sinU = 0.96

Step-by-step explanation:

sinU = [tex]\frac{opposite}{hypotenuse}[/tex] = [tex]\frac{WV}{UV}[/tex] = [tex]\frac{24}{25}[/tex] = 0.96

find the volume of the prism please

Answers

72
Step by step: 4*2*9=72

Answer:

the answer is 72

Step-by-step explanation:

when your finding volume use the formula length x width x height

Choose the number(s), if any, that make the equation or inequality true from the following set of numbers: {3, 4, 7, 9,12,18,32}.
number one:1/3f=4
number two:1/3 f < 4
number three: m + 7 =20
number 4: m + 7 ≥ 20

Answers

Answer:-2

Step-by-step explanation:

Other Questions
the mean of three number is 15. if 2 of the numbers are 18 and 16 what is the third number In the film "Amadeus" Salieri plays "Eine Kliene Nacht music" for the Priest. True ____ False____ Solve for x1 point68O68O 10 On January 1, Year 1, Michael sold a property with a remaining useful life of 20 years to Wei Co. for $800,000. On the same date, Michael leased back the property from Wei for 18 years. The lease was properly classified by Michael as a finance lease. Michael is not sure how to recognize the $800,000 received from Wei Co. on January 1, Year 1.Required:Which section of the Accounting Standards Codification best helps Michael Co. determine how the initial proceeds of $800,000 received from Wei Co. (buyer-lessor) are recognized? Whats the ara of each figure? Of the statistical processes we have covered throughout this course, which would be most helpful to determine if study hours and sleep could help estimate a student's test score? Can anyone help me solve this? The following functions give the populations of four towns with time tt in years.(i) P=600(1.12)^t(ii) P=1,000(1.03)^t(iii) P=200(1.08)^t (iv) P=900(0.90)^tWhich town has the largest percent growth rate? What is the percent growth rate? You receive a credit card application from Shady Banks Savings and Loan offering an introductory rate of .5 percent per year, compounded monthly for the first six months, increasing thereafter to 17.3 percent compounded monthly. Assume you transfer the $6,300 balance from your existing credit card and make no subsequent payments. How much interest will you owe at the end of the first year What will happen when the country'scurrency is undervalued? A scientist studied a population of birds for 10 years. During that time, the population was never fewer than 30 birds and never more than 50. Her data showed that over half of the birds born did not survive to reproduce, because of competition for food and predation. She noticed that In a single generation, about 90% of the birds that were born lived to reproduce, and the population increased to 90. Which inference(s) about this population might be true last question was wrong. Line l and line m are straight lines. What is the measure of angle y? Help someone plsss!!!!!!!!!! I need help with my math work . Which transitional words show cause and effect? Select 4 options.sothusfirstfinallythereforeconsequently i hat is the slope 6/-10 and 3/-13 sometimes balance point may not be obtained on the potentiometer wire why What is the uses of seawater as a source of renewable energy? If a normal distribution has a mean of 104 and a standard deviation of 6, what is the z-score for a value of 110?A. 1.5B. 2C. 1D. 0.5 What is the volume of the cone in the diagram?